37 votes
Accepted

Elegant recursion for A301897

Here is an expanded version of the generating function argument I sketched in a comment. For $i=1,2,3$, define the generating functions $F_i(x,y) := \sum_{n=0}^\infty \sum_{q=0}^\infty R(n,3q+i) x^n y^...
Terry Tao's user avatar
  • 109k
34 votes
Accepted

Can we just use the linear term of exponential sums to sum divergent series

This summation method gives answers that are close to, but do not always match, traditional divergent summation methods. For instance, for constants $a,b>0$, the divergent sum $$ \sum_{n=1}^\infty ...
Terry Tao's user avatar
  • 109k
34 votes
Accepted

Does this expression always vanish?

Recall that $$ \sum_iA_i^{n-1}\prod_{j\neq i}\frac1{A_i-A_j}=1, $$ as follows from the Lagrange interpolation of $x^{n-1}$. Now apply $\prod_i \frac \partial{\partial A_i}$. We get $$ \sum_i\left((...
Ilya Bogdanov's user avatar
33 votes
Accepted

Proving $\sum_{i=1}^{n}\sum_{j=1}^{n}\left\{\frac{x_{i}}{x_{j}}\right\}\le \frac{9}{14}n^2$?

OK, here is a fairly simple proof that for any positive integer $n$ and any positive real numbers $x_1,\ldots,x_n$, $$ \sum_{i,j=1}^{n}\left\{\frac{x_i}{x_j}\right\}\leq \frac{9}{14}n^2\,.$$ That the ...
fedja's user avatar
  • 59.8k
30 votes
Accepted

The Mompox Sequence: are all its terms different?

The answer is yes. Suppose $a(m)=a(n)$ for some $m<n$. By the definition of the sequence, we have $a(n)=a(m)\prod_{i=m+1}^ni^{s_i}$, where $s_i\in\{1,-1\}$, so that the above product is equal to $1$...
Wojowu's user avatar
  • 27.4k
22 votes

Closed form of an infinite series

Denote $c_n:={(-1)^n \frac{\Gamma(\frac{1}{3}+\frac{n}{3})}{\Gamma(1+\frac{n}{3})} \sin(\frac{2\pi n}{3})}$ the $n$-th term of the series. We have for all $k\ge0$ $$c_{3k}=0,$$ $$c_{3k+1}= (-1)^{k+1}\...
Pietro Majer's user avatar
  • 56.6k
21 votes

Proving $\sum_{i=1}^{n}\sum_{j=1}^{n}\left\{\frac{x_{i}}{x_{j}}\right\}\le \frac{9}{14}n^2$?

Too long for a comment. For $c>0$, the following two claims are equivalent. Claim 1. An inequality $$ \sum_{i=1}^{n}\sum_{j=1}^{n}\left\{\dfrac{x_{i}}{x_{j}}\right\}\leqslant cn^2$$ holds for all ...
Fedor Petrov's user avatar
21 votes

Is this formula expressing $\operatorname{erf} \left(\tfrac 1 {\sqrt 2}\right)$ as an infinite series already known?

Let $$f(x):=\sqrt{\pi } e^{x^2/4} \operatorname{erf}(x/2).$$ Then $$f'(x)=\tfrac12\, x f(x)+1.$$ So, by the Leibniz formula, for $m\ge1$, $$f^{(m+1)}(x)=\tfrac12\,x f^{(m)}(x)+\tfrac m2\,f^{(m-1)}(x)$$...
Iosif Pinelis's user avatar
18 votes
Accepted

When can a sum be re-signed to converge to any limit?

Let's rearrange so that $a_1\ge a_2\ge a_3\ge \ldots$. Then this works if and only if $a_n\le \sum_{k>n}a_k$ for all $n\ge 1$. To see this, let me also rephrase the problem as: Can we obtain all $0\...
Christian Remling's user avatar
17 votes

How many digits of $\sqrt{2}$ are known to date?

My (very limited) understanding of the paper Polynomial Factorization and Nonrandomness of Bits of Algebraic and Some Transcendental Numbers is that bit strings of algebraic numbers are not "...
Sam Nead's user avatar
  • 26.3k
17 votes
Accepted

How many digits of $\sqrt{2}$ are known to date?

In one sense, all of the base-2 digits (or, I guess, "bits") of $\sqrt{2}$ are known because we have a closed-form formula, according to the OEIS: $$\begin{align} a(n) &= \frac{1}{2} - \...
mhum's user avatar
  • 1,625
14 votes

Behavior of $\sum_{n=1}^{\infty} (\{n \xi \} - \frac{1}{2})$ for irrational number $\xi$

This is studied in detail in Sums of Fractional Parts of Integer Multiples of an Irrational The sum $$C(N)=\sum_{n=1}^{N} \bigl( \{ n \xi \} - \tfrac{1}{2}\bigr)$$ satisfies $|C(N)|>c\log N$ for ...
Carlo Beenakker's user avatar
14 votes

On the finite sum of reciprocal Fibonacci sequences

Let me sketch a proof that this identity holds for big enough $n$. In fact, we can show that $$\left(\sum_{k = n}^{2n} \frac{1}{F_{2k}}\right)^{-1} = F_{2n-1} + \frac{1}{\varphi \sqrt{5}} + o(1),$$ ...
Aleksei Kulikov's user avatar
13 votes
Accepted

On 12 cfracs: for Catalan's $K$, Gieseking's $\kappa$, and $\pi^2$, $\pi^3$, plus three for $\zeta(3)$ using Zagier's "six sporadic sequences"

We have $$C_2(-17,-6,-72)=-(5/8)L(\chi_{-3},2)$$ and $$C_2(10,3,-9)=(1/2)L(\chi_{-3},2)$$ so both are proportional to what you call Gieseking's constant but which is simply the value at 2 of the L ...
Henri Cohen's user avatar
  • 11.7k
12 votes

Probability of winning game whereby $T+1$ heads in a row of a coin flip is required to win where $T$ is the number of cumulative tails flipped

The probability of not winning is $$ \prod_{T=1}^\infty \left(1 - \frac1{2^T} \right) = \frac12 \frac34 \frac78 \frac{15}{16} \cdots = 0.28878809508660 \ldots ; $$ that's a well-known constant (equal ...
Noam D. Elkies's user avatar
12 votes
Accepted

If $f(x) = \sum_{n=0}^\infty a_n x^n$, then $\int_{-\infty}^\infty f(x^2) dx = \pi i a_{-\frac{1}{2}}$

Let $\phi(z)$ be analytic function defined on the half-plane $$H(\delta)=\{z\in \mathbb{C}: \operatorname{Re}z\ge -\delta\}$$ for $0<\delta<1$. Suppose that, for some $A<\pi$, $\phi$ ...
Dispersion's user avatar
12 votes
Accepted

Closed form of an infinite series

Q: Does the following infinite series have a closed form? It does, according to Mathematica: $$\sum_{n=1}^{\infty} {(-1)^n \frac{\Gamma(\frac{1}{3}+\frac{n}{3})}{\Gamma(1+\frac{n}{3})} \sin\left(\frac{...
Carlo Beenakker's user avatar
12 votes

How to show that $\log 2(1/2\log 2\log 4 + 1/3\log 3\log 6 + \dotsb) + 1/2\log 2 - 1/3\log 3 + 1/4\log 4 - \dotsb = 1/\log 2$

Observe that the left-hand side is the sum of two convergent series. Let $N\geq 2$ be an integer tending to infinity. Truncate the first series at the $N$-th term and the second series at the $2N$-th ...
GH from MO's user avatar
  • 99.1k
12 votes
Accepted

A conjectured series expression for the Riemann $\xi$-function and/or Completed L-series. Could this be proven?

Your conjecture is true, and follows trivially from the Poisson summation formula. For instance, use Theorem 9.4.2 in my book joint with K. Belabas "Numerical Algorithms for number theory", ...
Henri Cohen's user avatar
  • 11.7k
12 votes

Is this formula expressing $\operatorname{erf} \left(\tfrac 1 {\sqrt 2}\right)$ as an infinite series already known?

WolframAlpha returns this result promptly, so it is probably known or straightforward to show. See here.
GH from MO's user avatar
  • 99.1k
11 votes
Accepted

Does a conditionally convergent sum with random signs converge almost surely?

The answer is no, $\sum \epsilon_n a_n$ need not almost surely converge. For instance, with $a_n = \frac{(-1)^n}{\sqrt{n}}$, the random series $\sum \epsilon_n a_n$ converges with probability zero. ...
Sam Hopkins's user avatar
  • 22.9k
11 votes
Accepted

Suitable closed form for the A079501

Yes! $i$ is the first part in the composition $j + 1$ is the number of other parts in the composition $+1$ accounts for the case that there is only one part Summing over $i$ and $j$, we want to ...
1001's user avatar
  • 726
11 votes
Accepted

Finding $\sum_i x_i$ given $\{\sum_i x_i^{2n}\}_{n\in \mathbb{N}}$

If you plot $\log f_n$ versus $n$, with $f_n=\sum_i x_i^{2n}$, then the asymptotic slope for large $n$ will give you the largest of the $x_i$; subtracting that contribution from $f_n$ and repeating ...
Carlo Beenakker's user avatar
10 votes
Accepted

Remarkable recursions for the A204262

I can show the first identity $R(n,0) = f_{n+1,n+1}(0)$, as a consequence of the more general identity $$ R(n,q) = \frac{1}{(q+1)!} f_{n+q+1,n}(n+1)\tag{1}\label{1}$$ for $n,q \geq 0$. Indeed, note ...
Terry Tao's user avatar
  • 109k
10 votes
Accepted

Asymptotic for Ramanujan's $\tau$-function

While an asymptotic for $|\tau(n)|$ does not exist, there are many results that help us to nail down the order of $|\tau(n)|$. First, let us write $$\tau(n)=n^{\frac{11}{2}}f(n).$$ Also, let $d(n)$ ...
2734364041's user avatar
  • 5,059
10 votes
Accepted

How can I evaluate the following sum?

The corresponding infinite sum is related to the "incomplete theta function", $$\Theta_0(x,y)=\sum_{n\geq 0}x^ny^{n(n-1)/2}.$$ We have $$\sum_{n\geq 0}a^{n^2}=\Theta_0(a,a^2).$$ Accordingly, ...
Alexandre Eremenko's user avatar
10 votes
Accepted

Binomial series

$\newcommand\ep\varepsilon\newcommand\de\delta$Note that $$\sum_{k=0}^n k^a\binom nk=n^a2^n E\Big(\frac{X_n}n\Big)^a, \tag{1}\label{1}$$ where $X_n$ is a binomial random variable with parameters $n,1/...
Iosif Pinelis's user avatar
9 votes
Accepted

On the finite sum of reciprocal Fibonacci sequences

We need to prove, equivalently $$\frac1{F_{2n-1}+1}<\sum_{k=n}^{2n}\frac1{F_{2k}}\le \frac1{F_{2n-1}}, $$ that is, by the above expression for $F_{k}$, since $\beta=-\alpha^{-1}$, we need to check ...
Pietro Majer's user avatar
  • 56.6k
9 votes
Accepted

Let $(a_n)_{n\in N}=(1,2,3,4,6,8,9,12,\cdots)$ list the set$\{2^n3^m\mid m,n\in N\}$. Find $α$ such that $(a_n)\alpha\pmod1$ is not equidistributed

$\alpha =\sum_{n=1}^{\infty} \frac{1}{ 6^{100^n}}$ should do the trick. A positive proportion of numbers on your list are of the form $2^a 3^b$ for $a,b$ within a reasonable constant factor of each ...
Will Sawin's user avatar
  • 137k
9 votes

Asymptotic for Ramanujan's $\tau$-function

No, $\tau(n)$ fluctuates wildly, and it cannot be described in simpler terms. It is "irreducible arithmetic data", and we just love that. Same for its absolute value.
GH from MO's user avatar
  • 99.1k

Only top scored, non community-wiki answers of a minimum length are eligible